LSAT and Law School Admissions Forum

Get expert LSAT preparation and law school admissions advice from PowerScore Test Preparation.

User avatar
 Dave Killoran
PowerScore Staff
  • PowerScore Staff
  • Posts: 5853
  • Joined: Mar 25, 2011
|
#28733
Complete Question Explanation
(The complete setup for this game can be found here: lsat/viewtopic.php?f=179&p=88386#p88386)

The correct answer choice is (B)

The question stem establishes that I is the second valve opened. From the fifth rule, which states that "I is opened after K," we can determine that K must be the first valve opened:

  • K ..... I
    1 ..... 2 ..... 3 ..... 4 ..... 5 ..... 6 ..... 7 ..... 8

With K and I placed, the remaining variables appear in the following configuration:

  • P :longline:
    ..... .....H :longline: O :longline: L
    N :longline:


    Plus, G :longline: L, which I'd connect to the diagram above if I were drawing this out on paper (but can't do very easily with the limited drawing tools in this Forum). However, the above should be sufficient to get the basic idea across :-D
From the above we can make a few inferences immediately. For example, we can determine that L must be the last valve opened. P, N, or G must be the third valve opened (after K and I are opened first and second). G has the most freedom and must be carefully tracked because it can be opened anywhere from third to seventh.


  • K ..... I ..... P/N/G ..... ..... ..... ..... ..... .....L
    1 ..... 2 ..... 3 ..... 4 ..... 5 ..... 6 ..... 7 ..... 8

With K, I and L placed, the remaining variables appear in the following configuration, and all five will fill spaces 3 through 7:

  • P :longline:
    ..... .....H :longline: O
    N :longline:


    G?
The question stem is a Could Be True Except, which means that the four incorrect answers Could Be True, and the one correct answer Cannot Be True. Because incorrect answers Could Be True, my first suspicion is that any answer containing G will be incorrect. Why? Because G has a wide range of placement options, and could be in any of the open spaces (3-7) since it has no active restrictions in place ("no active restrictions" simply means that all of the rules involving G have been accounted for and G has met those restrictions. Thus, G can be placed in any spot and it will meet all of the rules that affect it, and those rules are "dead" for our purposes).

Answer choices (A) and (E) both contain G, and since G could be third or seventh, both of these answer choices are incorrect. Note how G is sent to the two endpoints—they are testing to see if you understand the range of movement G has here.

Answer choice (B): This is the correct answer. Both P and N must be opened before H, which means that in this question the earliest that H can be opened is fifth. Thus, H cannot be opened fourth, and this answer choice cannot occur and is therefore correct.

Answer choice (C): Note how the test makers attempt to take a "front end" variable (P) and push it backwards down the line. This is a standard test making tool they use, and it throws some people off because they feel that P should be more towards the front. Plus, this tests whether or not you understand that G could be opened earlier. The following hypothetical shows that P could be the fifth valve opened:

  • K - I - N - G - P - H - O - L
Answer choice (D): O can be the sixth valve opened (note how this answer takes a "later" variable and pushes it forward a bit). The following hypothetical shows that O could be the sixth valve opened:

  • K - I - P - N - H - O - G - L
Consequently, answer choice (D) could occur and is thus incorrect.
 wrjackson1
  • Posts: 22
  • Joined: Apr 02, 2018
|
#45797
Hi, I was hoping you could help me with this problem. Although I got the answer right, I'm a little worried my method of doing so could cause problems in the future. I essentially just took stock of my rules and tried to think of scenarios where they could be true. It worked out where I didn't burn through too much time, but I'm not sure if it would work on harder questions. Any suggestions?
 Malila Robinson
PowerScore Staff
  • PowerScore Staff
  • Posts: 296
  • Joined: Feb 01, 2018
|
#46040
Hi wrjackson1,
It is a little hard to know what you mean by that. Getting to the correct answer always involves the rules and generally will involve thinking of different scenarios for the rules. Can you give us an example of what you did here so that we can see whether it would work in a general sense?
-Malila

Get the most out of your LSAT Prep Plus subscription.

Analyze and track your performance with our Testing and Analytics Package.